Add week 1 recitation problems for phys-102.
[course.git] / latex / problems / Serway_and_Jewett_8 / problem23.10.tex
1 \begin{problem*}{23.10}
2 Two small metallic spheres, each of mass $m=0.200\U{g}$, are suspended
3 as pendulums by light strings of length $L$ as shown if Figure~P23.10.
4 The spheres are given the same electric charge of $7.20\U{nC}$, and
5 they come to equilibrium when each string is at an angle of
6 $\theta=5.00\dg$ with the vertical.  How long are the strings?
7 \begin{center}
8 \begin{asy}
9 import Mechanics;
10 import ElectroMag;
11
12 real theta = 20; // 5;  Exaggerated for clarity
13 real L = 2cm;
14
15 real x = L*Sin(theta);
16 real y = -L*Cos(theta);
17
18 draw((x,y)--(0,0)--(-x,y));
19 draw((0,y)--(0,0), dashed);
20 dot((0,0));
21
22 Angle t = Angle((0,y), (0,0), (x,y), "$\theta$");
23 t.draw();
24
25 Charge a = pCharge((-x,y));
26 Charge b = pCharge((x,y));
27 a.draw(); b.draw();
28 \end{asy}
29 \end{center}
30 \end{problem*}
31
32 \begin{solution}
33 By symmetry the two spheres will be at the same height, so the
34 distance between them is $2L\sin(\theta)$ and we can draw a free body
35 diagram for the right-hand sphere:
36 \begin{center}
37 \begin{asy}
38 import Mechanics;
39
40 real theta = 5;
41 real E_mag = 0.1cm;
42
43 Vector T = Vector((0,0), mag=E_mag/Sin(theta), dir=90+theta, "$T$");
44 Vector G = Vector((0,0), mag=E_mag/Tan(theta), dir=-90, "$mg$");
45 Vector E = Vector((0,0), mag=3*E_mag, dir=0, "$F_E$");
46
47 T.draw();
48 G.draw();
49 E.draw();
50 dot((0,0));
51 \end{asy}
52 \end{center}
53
54 Because the system is in equilibrium, the net force on the sphere must
55 be zero.
56 \begin{align}
57   0 &= \sum F_y = T\cos(\theta) - mg \\
58   T &= \frac{mg}{\cos(\theta)} \\
59   0 &= \sum F_x = \frac{kq^2}{[2L\sin(\theta)]^2} - T\sin(\theta) \\ 
60   \frac{kq^2}{[2L\sin(\theta)]^2} &= T\sin(\theta)
61     = \frac{mg}{\cos(\theta)}\sin(\theta) = mg\tan(\theta) \\
62   [2L\sin(\theta)]^2 &= \frac{kq^2}{mg\tan(\theta)} \\
63   2L\sin(\theta) &= \sqrt{\frac{kq^2}{mg\tan(\theta)}}
64     = q\sqrt{\frac{k}{mg\tan(\theta)}} \\
65   L &= \frac{q}{2\sin(\theta)}\sqrt{\frac{k}{mg\tan(\theta)}}
66     = \ans{29.9\U{cm}}
67 \end{align}
68 \end{solution}